Add week 1 recitation problems for phys-102.
[course.git] / latex / problems / Serway_and_Jewett_8 / problem23.11.tex
1 \begin{problem*}{23.11}
2 Two small beads having positive charges $q_1=3q$ and $q_2=q$ are fixed
3 at the opposite ends of a horizontal insulating rod of length
4 $d=1.50\U{m}$.  The bead with charge $q_1$ is at the origin.  As shown
5 in Figure~P23.11, a third small, charged bead is free to slide on the
6 rod.  \Part{a} At what position $x$ is the third bead in
7 equilibrium?  \Part{b} Can the equilibrium be stable?
8 \begin{center}
9 \begin{asy}
10 import Mechanics;
11 import ElectroMag;
12
13 real d = 2cm;
14 real q2oq1 = 1/3.;
15 real x = (1-sqrt(q2oq1))/(1-q2oq1) * d;
16
17 Charge q1 = aCharge((0,0), q=3, "$q_1$");
18 Charge q2 = aCharge((d,0), q=1, "$q_2$");
19 Charge q3 = aCharge((x,0), q=0, "$q_3$");
20
21 Distance dx = Distance((0,0), (x,0), offset=-30pt, "$x$");
22 Distance dd = Distance((0,0), (d,0), offset=-12pt, "$d$");
23
24 dx.draw();
25 dd.draw();
26 q1.draw();
27 q2.draw();
28 q3.draw();
29 \end{asy}
30 \end{center}
31 \end{problem*}
32
33 \begin{solution}
34 \Part{a}
35 The electric field along the rod due to the two end charges has a $y$
36 component of $0$ and an $x$ component of
37 \begin{equation}
38   E = \frac{kq_1}{r_1^2} - \frac{kq_2}{r_2^2}
39     = k\p({\frac{3q}{x^2} - \frac{q}{(d-x)^2}})
40     = kq\p({\frac{3}{x^2} - \frac{1}{(d-x)^2}})
41 \end{equation}
42 \begin{center}
43 \begin{asy}
44 import graph;
45
46 size(6cm,4cm,IgnoreAspect);
47
48 real q2oq1 = 1/3.;
49 real X = (1-sqrt(q2oq1))/(1-q2oq1);
50
51 real E(real x) { return 3./x^2 - 1./(1.-x)^2; }
52 real Z(real x) { return 0; }
53
54 real xmin = 0.15;
55 real xmax = 0.9;
56
57 draw((xmin,0)--(xmax,0), grey+dashed);
58 draw((X,0)--(X,E(xmax)), grey+dashed);
59 draw(graph(E, xmin, xmax), red);
60
61 xaxis("$x/d$", YEquals(E(xmax)), xmin=xmin, xmax=xmax, LeftTicks);
62 yaxis("$\frac{E}{kq}$", XEquals(xmin), ymin=E(xmax), ymax=E(xmin), RightTicks);
63 \end{asy}
64 \end{center}
65
66 The third bead will be in equilibrium when this electric field is
67 zero, which occurs when
68 \begin{align}
69   0 &= \frac{3}{x_0^2} - \frac{1}{(d-x_0)^2} \\
70   \frac{1}{(d-x_0)^2} &= \frac{3}{x_0^2} \\
71   x_0^2 &= 3(d^2 - 2dx_0 + x_0^2) \\
72   0 &= 2x_0^2 - 6 dx_0 + 3d^2 \\
73   0 &= 2\p({\frac{x_0}{d}})^2 - 6 \frac{x_0}{d} + 3 \\
74   \frac{x_0}{d} &= \frac{6 \pm \sqrt{(-6)^2 - 4\cdot2\cdot3}}{2\cdot2}
75     = 2.366 \text{ or } 0.6340 \\
76   x_0 &= \ans{0.951\U{m}} \;.
77 \end{align}
78 The second $x_0$ is greater than $d$, so it is non-physical.
79
80 \Part{b}
81 If the third bead has a positive charge, it will be pushed to the
82 right for $x<x_0$ ($E(x<x_0)>0$) and to the left for $x>x_0$
83 ($E(x>x_0)<0$), so it will be stable.  If it has a negative charge it
84 will not be stable.
85 \end{solution}